LSAT and Law School Admissions Forum

Get expert LSAT preparation and law school admissions advice from PowerScore Test Preparation.

 Administrator
PowerScore Staff
  • PowerScore Staff
  • Posts: 8917
  • Joined: Feb 02, 2011
|
#37016
Please post below with any questions!
 bridgetka
  • Posts: 2
  • Joined: Jul 10, 2017
|
#37115
The answer is D, but the reasoning seems a bit shaky to me. It seems that, with answer D, the test writers are trying to weaken the stimulus by suggesting that those individuals who watched themselves exercise might have just over-reported that they exercised more compared to those who watched other people on the treadmill. Maybe watching a recording of yourself doesn't actually motivate you to exercise and instead, just makes you exaggerate more than the control group.

Here's my question: watching your identical twin seems doesn't seem like it is analogous to watching yourself because, to state the obvious, if you are watching your twin, you are NOT watching yourself. Yes, you are watching someone that shares the same genetic makeup as you, but it is still not you and twins, even identical ones, can tell which person is which when they look at photos/videos. So it's hard for me to extrapolate from answer choice D that watching one's twin read is a similar scenario to watching yourself exercise.

What am I missing here?

I narrowed it down to C and D and chose C even though I didn't love the answer. But D seemed like a trap to me, so I went with C.
User avatar
 Jonathan Evans
PowerScore Staff
  • PowerScore Staff
  • Posts: 726
  • Joined: Jun 09, 2016
|
#37124
Hi, BridgetKa,

Great question, and this is in fact an LR question I have given some thought to since I have reviewed this latest LSAT. In a response to a student comment on our recent LR recap (http://blog.powerscore.com/lsat/june-20 ... ning-recap), I wrote in reference to this question:
There was a Weaken question that had to do with people's estimation of time spent exercising based on watching a video of *themselves* exercising. The trick was that the credited response involved people who watched a video of their *identical* twin exercising were likely to overestimate their performance.

This kind of apparent bait-and-switch that's actually not we've noted is a rather common ploy on harder LR questions. Having read the PS books, you're probably familiar with the idea of Shell Game problems. This is a situation of a "Shell Game that wasn't": the twin is not enough of an issue to prevent this answer from being the option that *most* weakens this argument.
There are a couple key points at play in this question:
  1. Note the question task: we must find what, if true, would most weaken the argument (emphasis mine). In other words, given the options available, what would harm the conclusion the most. The credited response might not be great; we might not even like it at all; it might only harm the conclusion a little bit; but if it harms the conclusion the most, then that's what we're stuck with.
  2. Pay close attention to the precise issue in the conclusion. What's the claim here? What evidence do we have? Why is this evidence insufficient? The credited response will likely be at least connected to a salient flaw in the argument.
Let's zero in on the claim/conclusion: People who watch recordings of themselves exercising exercise on average more than people who do not watch such recordings.

What evidence do we have? These people who watch these videos self-report this higher than average amount of exercise.

What's the problem here? Do we actually believe them? Is what they say true?

Now consider the task: we wish to harm the likelihood of this conclusion; we wish to find evidence that even though people who watch videos of themselves exercising might claim they exercise more, they actually don't.

Consider the answer choices:

(A): This is irrelevant. The kind of exercise performed is not the issue here. No such distinction is made in the argument, and this does nothing to harm the connection between self-reporting and actual behavior.

(B): If anything, this answer very slightly strengthens the argument, though it suffers from similar but worse issues than those of the credited response!

(C): This answer requires a couple additional assumptions to make it work. It is the classic "fill-in-the-blanks" incorrect answer choice. In this case, if we knew that those who watched a video of themselves exercising were in fact already highly motivated to exercise, then it wasn't the video that made the difference. Heck, I'd need to be pretty motivated to watch myself run on a treadmill [shudder]. However, note that we must introduce a missing and unwarranted assumption to make this answer work.

(D) This is the credited response. Yes, we are dealing with an identical twin here, but this answer is the only one that directly harms the conclusion and it does so by attacking the most pronounced flaw in this argument, that is, do people accurately self-report their exercise habits? The evidence provided in this answer choice, while qualified somewhat by the "identical twin," does provide some reason to believe that people may not accurately self-report such behavior.

(E) This couch-potato answer sounds even more dull than watching myself on a treadmill. Maybe they all just fell into a stupor watching themselves sitting on a couch. In any event, this answer if anything strengthens the conclusion marginally with its problematic couch-potato/exercise analogy.

I hope this helps!
 bridgetka
  • Posts: 2
  • Joined: Jul 10, 2017
|
#37160
Hmm...okay i think I get it. So it's about the fact that there could be some scenarios in the world in which people overreport their results and because that's true in some circumstances it could be true here? I guess I viewed answer choice D as a flawed argument by analogy answer choice, where the analogy didn't match the stimulus whatsoever. But i guess this isn't a flaw question, it's a weakening question so the correct answer choice doesn't need to COMPLETELY destroy the validity of the argument. It just has to make the stimulus less likely to be correct.

Thank you!
 bli2016
  • Posts: 67
  • Joined: Nov 29, 2016
|
#37320
Hi, I read the explanation above for why C is not the correct answer, but I still don't think I understand it. Is it because the participants who are already highly motivated to exercise don't matter? I.e., if those who are not already highly motivated reported exercising longer each day (considering their self-assessments are correct), then the conclusion is still valid?
User avatar
 Jonathan Evans
PowerScore Staff
  • PowerScore Staff
  • Posts: 726
  • Joined: Jun 09, 2016
|
#37655
Hi, Bli,

Good question! Let me clarify.

Answer Choice (C) requires several additional (and unwarranted) assumptions to make it conceivably weaken the conclusion.For instance, to make the statement in (C) weaken the conclusion, we must assume without providing justification that:
  • The first group with the longer exercisers comprised mostly "highly motivated" participants, an error in the integrity of the study.
  • Within this first group, the causal factor that contributed to the longer exercise was not "watching the video" but instead their already high degree of motivation.
  • Thus, watching the videos did not increase the motivation level. Instead, their existing higher level of motivation was the causal factor behind their increased amount of exercise.
Given these wholly unwarranted assumptions, we could imagine a way for (C) to weaken the conclusion. That's what I was referring to when I called it a "fill-in-the-blanks" answer: find a way to fit a square peg in a round hole by making unwarranted assumptions.

Do not do this.

Stick to the plain meaning of the text without trying to "tell a story" to make incorrect answers work. Instead, focus on improving predictions and recognizing the most significant flaw in questions like these so that you can anticipate the most likely way to harm or help a conclusion.

I hope this helps!
 bk1111
  • Posts: 103
  • Joined: Apr 22, 2017
|
#39302
Jonathan Evans wrote:Hi, Bli,

Good question! Let me clarify.

Answer Choice (C) requires several additional (and unwarranted) assumptions to make it conceivably weaken the conclusion.For instance, to make the statement in (C) weaken the conclusion, we must assume without providing justification that:
  • The first group with the longer exercisers comprised mostly "highly motivated" participants, an error in the integrity of the study.
  • Within this first group, the causal factor that contributed to the longer exercise was not "watching the video" but instead their already high degree of motivation.
  • Thus, watching the videos did not increase the motivation level. Instead, their existing higher level of motivation was the causal factor behind their increased amount of exercise.
Given these wholly unwarranted assumptions, we could imagine a way for (C) to weaken the conclusion. That's what I was referring to when I called it a "fill-in-the-blanks" answer: find a way to fit a square peg in a round hole by making unwarranted assumptions.

Do not do this.

Stick to the plain meaning of the text without trying to "tell a story" to make incorrect answers work. Instead, focus on improving predictions and recognizing the most significant flaw in questions like these so that you can anticipate the most likely way to harm or help a conclusion.

I hope this helps!
Hi - I perceived C as providing an alternate cause, thus weakening the conclusion. For example, it's not the videos that caused the first group to exercise more, but the fact that they were already motivated to exercise. Why is this approach to answer C incorrect? I didn't see it so much as an assumption as just an alternate way to explain the conclusion. Thanks
 James Finch
PowerScore Staff
  • PowerScore Staff
  • Posts: 943
  • Joined: Sep 06, 2017
|
#39836
Hi BK,

The problem with (C) is that we don't know anything about "highly motivated" participants. How many of them were there in the groups? Were they evenly distributed? Could they have skewed the data? What if the group sizes were 10,000, but there was only 1 highly motivated participant in each group? Without being able to answer these questions, we can't say if (C) actually weakens the conclusion or not, and so it cannot be the correct answer choice.

Compare this to (D), where we have people effectively watching themselves (their identical twin) doing something, then overreported the time they actually spent doing that thing. This suggests that the group that watched itself exercise in fact overreported how much they actually exercised.

Hope this helps!
 wrjackson1
  • Posts: 22
  • Joined: Apr 02, 2018
|
#44756
Hi, could you explain more the "fill-in-the-blank" assumptions? I understand it in this case, but I don't know if I would be able to apply it yet to another question with different context
 Francis O'Rourke
PowerScore Staff
  • PowerScore Staff
  • Posts: 471
  • Joined: Mar 10, 2017
|
#44923
Hi Jackson,

I believe that Jonathan gave a really good definition in his second response to this thread. You can define fill-in-the-blank answers as those that require you to provide "wholly unwarranted assumptions." That is, you go too far in your analysis and fill in some blanks that you believe make sense, but are not present in the argument and are not by commonsense true.

Usually I think of these answer choices as "so, what?" answers. That is, if I can't tell how the answer choice would impact the conclusion, I ask myself 'so how what that effect things?' If I find myself doing a lot of extra work by making additional assumptions not supported by the argument in order to make that answer choice affect the conclusion, then I will eliminate it and move on.

Question 16, answer choice (C) from the October 2003 LSAT is one of these. You can also find this on page 3-4 of the course book if you have that one handy. A lot of people will believe that this answer choice weakens the conclusion by making some strong assumptions that are simply not given in the stimulus.

Get the most out of your LSAT Prep Plus subscription.

Analyze and track your performance with our Testing and Analytics Package.